14 svar
85 visningar
naytte behöver inte mer hjälp
naytte Online 4618 – Moderator
Postad: 26 sep 14:37 Redigerad: 26 sep 14:42

Envariabelanalys - bevisa addititivet för gränsvärden

God eftermiddag!

I vår kurs i envariabelanalys har vi en bevislista med olika bevis vi ska kunna inför sluttentan. Jag håller nu på att gå igenom denna och försöka komma fram till bevisen själv i första hand. Just nu jobbar jag med följande bevis:

Jag har en lösning och jag undrar om den är korrekt. Den ser ut så här:

Givna är:

limxafx=L  (1)\displaystyle \lim_{x \to a} f\left(x\right)=L\;\;(1)

limxagx=M  (2)\displaystyle \lim_{x \to a} g\left(x\right)=M\;\;(2)

Det vi då ska visa är att:

ϵR>0(δR>0)(xDf)[x-a<δf(x)-L+g(x)-M<ϵ\displaystyle \left(\forall\epsilon\in\mathbb{R}_{>0}\right)(\exists\delta\in\mathbb{R}_{>0})(\forall x\in D_f)[\left|x-a\right|<\delta \implies \left[\left|f(x)-L+g(x)-M \right|<\epsilon \right]

Vi börjar med att införa en ny variabel ϵ'\epsilon' enligt ϵ=ϵ'/2\epsilon = \epsilon'/2. Vi kan då skriva om (1) och (2) enligt:

ϵR>0δR>0xDfx-a<δf(x)-L<ϵ'2  1\displaystyle \left(\forall\epsilon\in\mathbb{R}_{>0}\right)\left(\exists\delta\in\mathbb{R}_{>0}\right)\left(\forall x\in D_f\right)\left[\left|x-a\right|<\delta \implies \left|f(x)-L\right|<\frac{\epsilon'}{2}\right]\;\;\left(1 \right)

ϵR>0δR>0xDf[x-a<δg(x)-M<ϵ'2]  2\displaystyle \left(\forall\epsilon\in\mathbb{R}_{>0}\right)\left(\exists\delta\in\mathbb{R}_{>0}\right)\left(\forall x\in D_f \right)[\left|x-a\right|<\delta \implies \left|g(x)-M\right|<\frac{\epsilon'}{2}]\;\; \left(2 \right)

Detta borde medföra att:

ϵR>0δR>0xDf[x-a<δf(x)-L+g(x)-M<ϵ']  (3)\displaystyle \left(\forall\epsilon\in\mathbb{R}_{>0}\right)\left(\exists\delta\in\mathbb{R}_{>0}\right)\left(\forall x\in D_f \right)[\left|x-a\right|<\delta \implies \left|f(x)-L\right|+\left|g(x)-M\right|<\epsilon'] \;\;(3)

Med hjälp av triangelolikheten kan vi sedan dra slutsatsen att:

f(x)-L+g(x)-Mf(x)-L+g(x)-M<ϵ'f(x)-L+g(x)-M<ϵ'\displaystyle \left|f(x)-L+g(x)-M\right|\le \left|f(x)-L\right|+\left|g(x)-M\right|<\epsilon'\implies \left|f(x)-L+g(x)-M\right|<\epsilon'

Eftersom ϵ\epsilon kan väljas godtyckligt, kan även ϵ'\epsilon' väljas godtyckligt. Således har vi att:

ϵR>0(δR>0)(xDf)[x-a<δ[f(x)-L+g(x)-M<ϵ]\displaystyle \left(\forall\epsilon\in\mathbb{R}_{>0}\right)(\exists\delta\in\mathbb{R}_{>0})(\forall x\in D_f)[\left|x-a\right|<\delta \implies [\left|f(x)-L+g(x)-M \right|<\epsilon]

\blacksquare

Gustor 150
Postad: 26 sep 15:03 Redigerad: 26 sep 15:05

Rackarns, min telefon dog precis innan jag skulle posta mitt svar...

När du säger "borde medföra att", så är det lite oklart. Du vet inte a priori att du kan hitta ett sådant delta för f(x)+g(x), utan det du ska bevisa är att givet något fixt epsilon, så finns det ett delta sådant att...

Notera att för ett givet epsilon, så kan du hitta delta_1 sådant att |x-x0|<delta_1 medför |f(x)-L|<epsilon, men detta delta_1 är inte nödvändigtvis samma för g(x) och samma epsilon.

Visa spoiler

Använd definitionen på f och g där du hittar delta_1 och delta_2 för värdet epsilon/2. Definiera sedan ett delta utifrån dessa.

naytte Online 4618 – Moderator
Postad: 26 sep 15:28 Redigerad: 26 sep 15:28

Borde jag inte bara kunna välja det δ\delta som är mindre av de två? Om jag vet att |x-a|<δ1|f(x)-L|<ϵ\displaystyle |x-a|<\delta_1 \implies |f(x)-L|<\epsilon samt att δ2<δ1\delta_2 < \delta_1, så borde det väl också gälla att |x-a|<δ2|f(x)-L|<ϵ\displaystyle |x-a|<\delta_2 \implies |f(x)-L|<\epsilon?

Gustor 150
Postad: 26 sep 15:39

Testa!

naytte Online 4618 – Moderator
Postad: 26 sep 16:22 Redigerad: 26 sep 16:25

När jag testar numeriskt verkar det stämma. Låt säga att vi undersöker gränsvärdet i punkten x=2x=2 för funktionen fx=x2f\left(x\right) = x^2 och att vi väljer ϵ=1/2\epsilon = 1/2. Då måste vi hitta ett δ\delta sådant att |x2-4|<1/2|x^2-4| < 1/2. Om vi väljer t.ex. δ=1/100\delta = 1/100 funkar det. Och vi kan ju alltid välja ett mindre och mindre δ\delta, och fortfarande ha att det medför |x2-4|<1/2|x^2-4| < 1/2. Så om man hittar ett δ\delta för ett givet ϵ\epsilon, kan man alltid göra sitt δ\delta mindre.

Stämmer den slutsatsen?

Rent grafiskt vill vi ju ligga på kurvan mellan dessa horisontella gränser:

Och det kan man ju göra med oändligt många δ\delta under en viss gräns.

Gustor 150
Postad: 26 sep 16:36

Ja, eftersom |x-x0| < d ==> |x-x0| < d' för alla d' >= d. Men det jag tänkte på var att givet ett epsilon kan du hitta d1 och d2 sådana att |f-L|<epsilon/2 för det första, och |g-M|<epsilon/2 för det andra. Och så kan du ju ta d = min{d1, d2} och dra slutsatsen att när |x-x0|<d, så är |f(x) - L| + |g(x) - M| < epsilon/2 + epsilon/2. Det var så jag tänkte.

Kan tyvärr inte skriva riktiga ekvationer från telefonen, hoppas det är läsbart ändå.

naytte Online 4618 – Moderator
Postad: 26 sep 16:38 Redigerad: 26 sep 16:38

Och så kan du ju ta d = min{d1, d2}

Jo precis, om jag förstår notationen rätt var det precis dit jag ville komma. Det vi säger här är väl i princip att vi väljer ut det minsta av δ1\delta_1 och δ2\delta_2 och sätter det som vårt nya δ\delta?

Gustor 150
Postad: 26 sep 17:34

Japp, för det mindre av de två så gäller båda olikheterna så då kan du köra triangelolikheten och avsluta beviset.

naytte Online 4618 – Moderator
Postad: 27 sep 14:00 Redigerad: 27 sep 14:12

Okej, så ett nytt försök:

Låt ϵR>0\epsilon\in\mathbb{R}_{>0} och låt ϵ=ϵ'/2\epsilon = \epsilon'/2. Vi har således att:

x-a<δ1fx-L<ϵ'/2  (1)\displaystyle \left|x-a\right|<\delta_1 \implies \left|f\left(x\right)-L\right|<\epsilon'/2\;\;(1)

x-a<δ2gx-M<ϵ'/2  (2)\displaystyle \left|x-a\right|<\delta_2 \implies \left|g\left(x\right)-M\right|<\epsilon'/2\;\;(2)

Låt δ=min{δ1,δ2}\displaystyle \delta = \min\{ \delta_1,\delta_2 \}

Detta ger sammantaget (3)(3):

x-a<δfx-L+gx-Mfx-L+gx-M<ϵ'  (3)\displaystyle \left|x-a\right|<\delta \implies \left|f\left(x\right)-L+g\left(x\right)-M\right| \le \left|f\left(x\right)-L\right|+\left|g\left(x\right)-M\right|<\epsilon' \;\;(3)

Detta innebär med andra ord att:

x-a<δfx-L+gx-M<ϵ'\displaystyle \displaystyle \left|x-a\right|<\delta \implies \left|f\left(x\right)-L+g\left(x\right)-M\right| <\epsilon'

Eftersom ϵ\epsilon kan väljas godtyckligt kan även ϵ'\epsilon' väljas godtyckligt. Således har vi utifrån (1)(1) och (2)(2) att:

ϵR>0δR>0xDx-a<δfx-L+gx-M<ϵ\displaystyle \left(\forall \epsilon\in\mathbb{R}_{>0}\right)\left(\exists\delta\in\mathbb{R}_{>0}\right)\left(\forall x\in D\right)\left[\displaystyle \left|x-a\right|<\delta \implies \left|f\left(x\right)-L+g\left(x\right)-M\right| <\epsilon\right]

\blacksquare


Ser det bra ut nu? :D

Gustor 150
Postad: 27 sep 14:33 Redigerad: 27 sep 14:35

Ja, fast du behöver inte blanda in en ny variabel ε'. Enligt definitionen av gränsvärde kan du hitta ett delta för varje positivt reellt tal. Så i detta fall använder vi definitionen på f och g var och en, där vi väljer vi talet ε/2 och får då två olika delta. Det är helt korrekt det du skrivit, det är bara att du slipper argumentet att "eftersom ε kan väljas godtyckligt, kan även ε' väljas godtyckligt" om du gör så.

Hur ska man egentligen hantera x=a? Hur blir det när man gör som du nu, dvs typ ignorerar det?

Annars borde man väl sätta 0<abs(x-a)<delta alternativt ta ut a från (D\{a})?

Vår föreläsare tar ut a från D och i kursboken (Analys i en variabel. Persson och Böiers.) pratar de om lite olika varianter men förstår inte riktigt skillnaden. Har ni några tankar?

naytte Online 4618 – Moderator
Postad: 27 sep 19:34 Redigerad: 27 sep 19:35

alternativt ta ut a från (D\{a})?

Japp, i mitt huvud hade jag redan gjort det men tydligen inte på pappret.

Det som gör det lite förvirrande ibland är att ϵ\epsilon-δ\delta-kriteriet för kontinuitet är identiskt men utan kravet att xax≠a. Så ibland skriver man på rent muskelminne och då blir det så här!

Gustor 150
Postad: 27 sep 19:37 Redigerad: 27 sep 19:38

Ja, det är egentligen rimligare att utesluta x=a ur definitionen som du skriver. Annars antar man att funktionen är kontinuerlig i x=a, vilket inte behöver vara fallet. Till exempel behöver f inte vara definierad i x=a även om gränsvärdet existerar.

Okaj. Och att ta ut från D eller sätta avståndet till a strikt större än 0 är samma sak?

Ja.

Svara
Close